Merge remote branch 'public/master'
[course.git] / latex / problems / Serway_and_Jewett_4 / problem08.24.tex
1 \begin{problem*}{8.24}
2 An $m_1 = 90\U{kg}$ fullback running east (\ihat) with a speed of $v_1
3 = 5.00\U{m/s}$ is tackled by an $m_2 = 95\U{kg}$ opponent running
4 north (\jhat) with a speed of $v_2 = 3.00\U{m/s}$.  Noting that the
5 collision is perfectly inelastic,
6
7  \Part{a} calculate the speed $v_f$ and direction $\theta$ of the
8  players just after the tackle and
9  \Part{b} determine the mechanical energy lost as a result of the
10  collision.  Account for the missing energy.
11 \end{problem*} % problem 8.24
12
13 \begin{solution}
14 \Part{a}
15 Conserving momentum in the \ihat\ and \jhat\ directions
16 \begin{align}
17  P_{ix} = m_1 v_1 &= P_{fx} = (m_1 + m_2) v_{fx} \\
18   v_{fx} &= v_1 \frac{m_1}{m_1 + m_2} = 2.43\U{m/s} \\
19  P_{iy} = m_2 v_2 &= P_{fy} = (m_1 + m_2) v_{fy} \\
20   v_{fy} &= v_2 \frac{m_2}{m_1 + m_2} = 1.54\U{m/s} \\
21  v_f &= \sqrt{v_{fx}^2 + v_{fy}^2} = \ans{2.88\U{m/s}} \\
22  \theta &= \arctan\left(\frac{v_{fy}}{v_{fx}}\right) = \ans{32.3\dg}
23 \end{align}
24
25 \Part{b}
26 \begin{align}
27  \Delta K &= K_f - K_i = \left( \frac{1}{2} m_1 v_1^2 + \frac{1}{2} m_2 v_2^2 \right) - \frac{1}{2} (m_1 + m_2) v_f^2 \\
28           &= \frac{1}{2} \left[ 90.0\U{kg} (5.00\U{m.s})^2 + 95.0\U{kg} (3.00\U{kg})^2 - (90.0\U{kg} + 95.0\U{kg}) (2.88\U{m/s})^2 \right]\\
29           &= \ans{-786\U{J}}
30 \end{align}
31 All of which has been lost as mechanical energy, and is now thermal
32 energy (warmer football players), noise (a loud crunch), etc.
33 \end{solution}